LSAT and Law School Admissions Forum

Get expert LSAT preparation and law school admissions advice from PowerScore Test Preparation.

 Administrator
PowerScore Staff
  • PowerScore Staff
  • Posts: 8917
  • Joined: Feb 02, 2011
|
#32004
Complete Question Explanation
(The complete setup for this game can be found here: lsat/viewtopic.php?t=15601)

The correct answer choice is (D)

If N is visited on Thursday we have two possibilities: either O is out or L is out (these are given as Templates 1 and 2b). Let’s quickly explore both:

If N is visited on Thursday and O is not visited (Template 1), then L must be visited on Wednesday. To ensure that N and S are not visited on consecutive days, S cannot be visited on Friday. And neither can H, in accordance with the first rule. Therefore, P must be visited on Friday:
PT79_Game_#1_#5_diagram 1.png
On the other hand, if N is visited on Thursday and L is not visited (Template 2b), we need to accommodate the OH block while also ensuring that N and S are not visited on consecutive days. To do so, we need to place the OH block on Tuesday and Wednesday, and S on Monday. Once again, P is the only neighborhood left to be visited on Friday:
PT79_Game_#1_#5_diagram 2.png
To answer this Must Be True question, we need to determine a consistency between our two solutions. And indeed, in both of them P is visited on Friday—an inference that quickly validates answer choice (D).
You do not have the required permissions to view the files attached to this post.

Get the most out of your LSAT Prep Plus subscription.

Analyze and track your performance with our Testing and Analytics Package.